Derive error term by using Taylor series expansions.












4












$begingroup$



Using Taylor series expansions, derive the error term for the formula
begin{equation}
f''(x)approx frac{1}{h^{2}}left [ f(x)-2f(x+h)+f(x+2h) right ].
end{equation}




I've tried it on my own way. We see that
begin{align*}
f(x+h)&=sum_{k=0}^{3}frac{h^{k}}{k!}f^{(k)}(x)+E_{n}(h)\
&=f(x)+hf'(x)+frac{h^{2}}{2}f''(x)+frac{h^{3}}{6}f'''(x)+E_{3}(h)
end{align*}
begin{align*}
f(x+2h)&=sum_{k=0}^{3}frac{(2h)^{k}}{k!}f^{(k)}(x)+E_{n}(2h)\
&=f(x)+2hf'(x)+2h^{2}f''(x)+frac{4h^{3}}{3}f'''(x)+E_{3}(2h)
end{align*}
and
begin{equation}
f(x+2h)-2f(x+h)=-f(x)+h^{2}f''(x)+h^{3}f'''(x)+E_{3}(2h)-E_{3}(h)
end{equation}
then by isolating $f''(x)$ we get
begin{equation}
f''(x)=frac{1}{h^{2}}left [ f(x+2h)-2f(x+h)+f(x) right ]-hf'''(x)-frac{1}{h^{2}}left [E_{3}(2h)-E_{3}(h) right ]
end{equation}
which isn't the right way to do since the term of "$-hf'''(x)$" is added (that's not mentioned in this problem). The rest is the error term. How do I answer it correctly?










share|cite|improve this question











$endgroup$








  • 1




    $begingroup$
    Maybe it is a dumb question, but why don't you just stop at the $E_2$ terms?
    $endgroup$
    – Ian Mateus
    Oct 14 '13 at 0:24










  • $begingroup$
    $-(1/h^2) (E_3(2h) - E_3(h))$ isn't explicitly mentioned in this problem either.
    $endgroup$
    – Hurkyl
    Oct 14 '13 at 0:45












  • $begingroup$
    @IanMateus Thanks for your answer. It works!
    $endgroup$
    – UnknownW
    Oct 14 '13 at 1:23






  • 1




    $begingroup$
    @AjmalW: Right: but it's not the error term. The error term for the approximation you're proving $$-hf'''(x) - (1/h^2)(E_3(2h) - E_3(h))$$ (actually, I think your work is missing some factors of $2$, and that should probably be $2E_3(h)$)
    $endgroup$
    – Hurkyl
    Oct 14 '13 at 2:16








  • 1




    $begingroup$
    I suppose the confusion is over what the phrase "error term" means. In general, if we approximate $f$ with $g$, the error term to the approximation means $g - f$. $E_3(x,h)$, for example, is the error term when approximating $f(x)$ with its third order Taylor series. The error term for the approximation you're proving is not "the terms involving $E_3$", but instead "the difference between the two sides of the approximation".
    $endgroup$
    – Hurkyl
    Oct 14 '13 at 2:22


















4












$begingroup$



Using Taylor series expansions, derive the error term for the formula
begin{equation}
f''(x)approx frac{1}{h^{2}}left [ f(x)-2f(x+h)+f(x+2h) right ].
end{equation}




I've tried it on my own way. We see that
begin{align*}
f(x+h)&=sum_{k=0}^{3}frac{h^{k}}{k!}f^{(k)}(x)+E_{n}(h)\
&=f(x)+hf'(x)+frac{h^{2}}{2}f''(x)+frac{h^{3}}{6}f'''(x)+E_{3}(h)
end{align*}
begin{align*}
f(x+2h)&=sum_{k=0}^{3}frac{(2h)^{k}}{k!}f^{(k)}(x)+E_{n}(2h)\
&=f(x)+2hf'(x)+2h^{2}f''(x)+frac{4h^{3}}{3}f'''(x)+E_{3}(2h)
end{align*}
and
begin{equation}
f(x+2h)-2f(x+h)=-f(x)+h^{2}f''(x)+h^{3}f'''(x)+E_{3}(2h)-E_{3}(h)
end{equation}
then by isolating $f''(x)$ we get
begin{equation}
f''(x)=frac{1}{h^{2}}left [ f(x+2h)-2f(x+h)+f(x) right ]-hf'''(x)-frac{1}{h^{2}}left [E_{3}(2h)-E_{3}(h) right ]
end{equation}
which isn't the right way to do since the term of "$-hf'''(x)$" is added (that's not mentioned in this problem). The rest is the error term. How do I answer it correctly?










share|cite|improve this question











$endgroup$








  • 1




    $begingroup$
    Maybe it is a dumb question, but why don't you just stop at the $E_2$ terms?
    $endgroup$
    – Ian Mateus
    Oct 14 '13 at 0:24










  • $begingroup$
    $-(1/h^2) (E_3(2h) - E_3(h))$ isn't explicitly mentioned in this problem either.
    $endgroup$
    – Hurkyl
    Oct 14 '13 at 0:45












  • $begingroup$
    @IanMateus Thanks for your answer. It works!
    $endgroup$
    – UnknownW
    Oct 14 '13 at 1:23






  • 1




    $begingroup$
    @AjmalW: Right: but it's not the error term. The error term for the approximation you're proving $$-hf'''(x) - (1/h^2)(E_3(2h) - E_3(h))$$ (actually, I think your work is missing some factors of $2$, and that should probably be $2E_3(h)$)
    $endgroup$
    – Hurkyl
    Oct 14 '13 at 2:16








  • 1




    $begingroup$
    I suppose the confusion is over what the phrase "error term" means. In general, if we approximate $f$ with $g$, the error term to the approximation means $g - f$. $E_3(x,h)$, for example, is the error term when approximating $f(x)$ with its third order Taylor series. The error term for the approximation you're proving is not "the terms involving $E_3$", but instead "the difference between the two sides of the approximation".
    $endgroup$
    – Hurkyl
    Oct 14 '13 at 2:22
















4












4








4


2



$begingroup$



Using Taylor series expansions, derive the error term for the formula
begin{equation}
f''(x)approx frac{1}{h^{2}}left [ f(x)-2f(x+h)+f(x+2h) right ].
end{equation}




I've tried it on my own way. We see that
begin{align*}
f(x+h)&=sum_{k=0}^{3}frac{h^{k}}{k!}f^{(k)}(x)+E_{n}(h)\
&=f(x)+hf'(x)+frac{h^{2}}{2}f''(x)+frac{h^{3}}{6}f'''(x)+E_{3}(h)
end{align*}
begin{align*}
f(x+2h)&=sum_{k=0}^{3}frac{(2h)^{k}}{k!}f^{(k)}(x)+E_{n}(2h)\
&=f(x)+2hf'(x)+2h^{2}f''(x)+frac{4h^{3}}{3}f'''(x)+E_{3}(2h)
end{align*}
and
begin{equation}
f(x+2h)-2f(x+h)=-f(x)+h^{2}f''(x)+h^{3}f'''(x)+E_{3}(2h)-E_{3}(h)
end{equation}
then by isolating $f''(x)$ we get
begin{equation}
f''(x)=frac{1}{h^{2}}left [ f(x+2h)-2f(x+h)+f(x) right ]-hf'''(x)-frac{1}{h^{2}}left [E_{3}(2h)-E_{3}(h) right ]
end{equation}
which isn't the right way to do since the term of "$-hf'''(x)$" is added (that's not mentioned in this problem). The rest is the error term. How do I answer it correctly?










share|cite|improve this question











$endgroup$





Using Taylor series expansions, derive the error term for the formula
begin{equation}
f''(x)approx frac{1}{h^{2}}left [ f(x)-2f(x+h)+f(x+2h) right ].
end{equation}




I've tried it on my own way. We see that
begin{align*}
f(x+h)&=sum_{k=0}^{3}frac{h^{k}}{k!}f^{(k)}(x)+E_{n}(h)\
&=f(x)+hf'(x)+frac{h^{2}}{2}f''(x)+frac{h^{3}}{6}f'''(x)+E_{3}(h)
end{align*}
begin{align*}
f(x+2h)&=sum_{k=0}^{3}frac{(2h)^{k}}{k!}f^{(k)}(x)+E_{n}(2h)\
&=f(x)+2hf'(x)+2h^{2}f''(x)+frac{4h^{3}}{3}f'''(x)+E_{3}(2h)
end{align*}
and
begin{equation}
f(x+2h)-2f(x+h)=-f(x)+h^{2}f''(x)+h^{3}f'''(x)+E_{3}(2h)-E_{3}(h)
end{equation}
then by isolating $f''(x)$ we get
begin{equation}
f''(x)=frac{1}{h^{2}}left [ f(x+2h)-2f(x+h)+f(x) right ]-hf'''(x)-frac{1}{h^{2}}left [E_{3}(2h)-E_{3}(h) right ]
end{equation}
which isn't the right way to do since the term of "$-hf'''(x)$" is added (that's not mentioned in this problem). The rest is the error term. How do I answer it correctly?







numerical-methods






share|cite|improve this question















share|cite|improve this question













share|cite|improve this question




share|cite|improve this question








edited Oct 14 '13 at 0:20







UnknownW

















asked Oct 14 '13 at 0:04









UnknownWUnknownW

985822




985822








  • 1




    $begingroup$
    Maybe it is a dumb question, but why don't you just stop at the $E_2$ terms?
    $endgroup$
    – Ian Mateus
    Oct 14 '13 at 0:24










  • $begingroup$
    $-(1/h^2) (E_3(2h) - E_3(h))$ isn't explicitly mentioned in this problem either.
    $endgroup$
    – Hurkyl
    Oct 14 '13 at 0:45












  • $begingroup$
    @IanMateus Thanks for your answer. It works!
    $endgroup$
    – UnknownW
    Oct 14 '13 at 1:23






  • 1




    $begingroup$
    @AjmalW: Right: but it's not the error term. The error term for the approximation you're proving $$-hf'''(x) - (1/h^2)(E_3(2h) - E_3(h))$$ (actually, I think your work is missing some factors of $2$, and that should probably be $2E_3(h)$)
    $endgroup$
    – Hurkyl
    Oct 14 '13 at 2:16








  • 1




    $begingroup$
    I suppose the confusion is over what the phrase "error term" means. In general, if we approximate $f$ with $g$, the error term to the approximation means $g - f$. $E_3(x,h)$, for example, is the error term when approximating $f(x)$ with its third order Taylor series. The error term for the approximation you're proving is not "the terms involving $E_3$", but instead "the difference between the two sides of the approximation".
    $endgroup$
    – Hurkyl
    Oct 14 '13 at 2:22
















  • 1




    $begingroup$
    Maybe it is a dumb question, but why don't you just stop at the $E_2$ terms?
    $endgroup$
    – Ian Mateus
    Oct 14 '13 at 0:24










  • $begingroup$
    $-(1/h^2) (E_3(2h) - E_3(h))$ isn't explicitly mentioned in this problem either.
    $endgroup$
    – Hurkyl
    Oct 14 '13 at 0:45












  • $begingroup$
    @IanMateus Thanks for your answer. It works!
    $endgroup$
    – UnknownW
    Oct 14 '13 at 1:23






  • 1




    $begingroup$
    @AjmalW: Right: but it's not the error term. The error term for the approximation you're proving $$-hf'''(x) - (1/h^2)(E_3(2h) - E_3(h))$$ (actually, I think your work is missing some factors of $2$, and that should probably be $2E_3(h)$)
    $endgroup$
    – Hurkyl
    Oct 14 '13 at 2:16








  • 1




    $begingroup$
    I suppose the confusion is over what the phrase "error term" means. In general, if we approximate $f$ with $g$, the error term to the approximation means $g - f$. $E_3(x,h)$, for example, is the error term when approximating $f(x)$ with its third order Taylor series. The error term for the approximation you're proving is not "the terms involving $E_3$", but instead "the difference between the two sides of the approximation".
    $endgroup$
    – Hurkyl
    Oct 14 '13 at 2:22










1




1




$begingroup$
Maybe it is a dumb question, but why don't you just stop at the $E_2$ terms?
$endgroup$
– Ian Mateus
Oct 14 '13 at 0:24




$begingroup$
Maybe it is a dumb question, but why don't you just stop at the $E_2$ terms?
$endgroup$
– Ian Mateus
Oct 14 '13 at 0:24












$begingroup$
$-(1/h^2) (E_3(2h) - E_3(h))$ isn't explicitly mentioned in this problem either.
$endgroup$
– Hurkyl
Oct 14 '13 at 0:45






$begingroup$
$-(1/h^2) (E_3(2h) - E_3(h))$ isn't explicitly mentioned in this problem either.
$endgroup$
– Hurkyl
Oct 14 '13 at 0:45














$begingroup$
@IanMateus Thanks for your answer. It works!
$endgroup$
– UnknownW
Oct 14 '13 at 1:23




$begingroup$
@IanMateus Thanks for your answer. It works!
$endgroup$
– UnknownW
Oct 14 '13 at 1:23




1




1




$begingroup$
@AjmalW: Right: but it's not the error term. The error term for the approximation you're proving $$-hf'''(x) - (1/h^2)(E_3(2h) - E_3(h))$$ (actually, I think your work is missing some factors of $2$, and that should probably be $2E_3(h)$)
$endgroup$
– Hurkyl
Oct 14 '13 at 2:16






$begingroup$
@AjmalW: Right: but it's not the error term. The error term for the approximation you're proving $$-hf'''(x) - (1/h^2)(E_3(2h) - E_3(h))$$ (actually, I think your work is missing some factors of $2$, and that should probably be $2E_3(h)$)
$endgroup$
– Hurkyl
Oct 14 '13 at 2:16






1




1




$begingroup$
I suppose the confusion is over what the phrase "error term" means. In general, if we approximate $f$ with $g$, the error term to the approximation means $g - f$. $E_3(x,h)$, for example, is the error term when approximating $f(x)$ with its third order Taylor series. The error term for the approximation you're proving is not "the terms involving $E_3$", but instead "the difference between the two sides of the approximation".
$endgroup$
– Hurkyl
Oct 14 '13 at 2:22






$begingroup$
I suppose the confusion is over what the phrase "error term" means. In general, if we approximate $f$ with $g$, the error term to the approximation means $g - f$. $E_3(x,h)$, for example, is the error term when approximating $f(x)$ with its third order Taylor series. The error term for the approximation you're proving is not "the terms involving $E_3$", but instead "the difference between the two sides of the approximation".
$endgroup$
– Hurkyl
Oct 14 '13 at 2:22












2 Answers
2






active

oldest

votes


















1












$begingroup$

With the purpose of not leaving this question unanswered, this answer contains what OP already knows: stopping at $E_2$ is the cleanest way to go, since we already know how to express the error term of a Taylor series in convenient ways (Lagrange form, integral form). Some algebra quickly yields the error $h^{-2}(E_2(2h)-2E_2(h))$.






share|cite|improve this answer









$endgroup$





















    0












    $begingroup$

    As a central difference quotient for the position $x+h$, the Taylor expansion of the expression starts as
    $$
    frac1{h^{2}}[f(x)-2f(x+h)+f(x+2h)]=f''(x+h)+frac{h^2}{12}f^{(4)}(x+h)+O(h^4). tag{*}label{eq:1}
    $$



    By Taylor expansion of the second derivative $g(x)=f''(x)$ one gets that the deviation from $x$ to $x+h$ expands as
    begin{align}
    f''(x+h)=g(x+h)&=g(x)+g'(x)h+frac{h^2}2g''(x)+O(h^3)
    \
    &=f''(x)+hf'''(x)+frac{h^2}2f^{(4)}(x)+O(h^2). tag{**}label{eq:2}
    end{align}



    Inserting eqref{eq:2} into eqref{eq:1} gives in total
    $$
    frac1{h^{2}}[f(x)-2f(x+h)+f(x+2h)]=f''(x)+hf'''(x)+frac{7h^2}{12}f^{(4)}(x)+O(h^3),
    $$

    so that the error term from the question should be
    $$
    -hf'''(x)-frac{7h^2}{12}f^{(4)}(x)+O(h^3).
    $$

    Obviously the linear error term is the dominant one. So up to the missing $2$ in front of $E_3(h)=h^4R_3(h)$ as mentioned in the comments, the calculation is correct.






    share|cite|improve this answer











    $endgroup$













      Your Answer





      StackExchange.ifUsing("editor", function () {
      return StackExchange.using("mathjaxEditing", function () {
      StackExchange.MarkdownEditor.creationCallbacks.add(function (editor, postfix) {
      StackExchange.mathjaxEditing.prepareWmdForMathJax(editor, postfix, [["$", "$"], ["\\(","\\)"]]);
      });
      });
      }, "mathjax-editing");

      StackExchange.ready(function() {
      var channelOptions = {
      tags: "".split(" "),
      id: "69"
      };
      initTagRenderer("".split(" "), "".split(" "), channelOptions);

      StackExchange.using("externalEditor", function() {
      // Have to fire editor after snippets, if snippets enabled
      if (StackExchange.settings.snippets.snippetsEnabled) {
      StackExchange.using("snippets", function() {
      createEditor();
      });
      }
      else {
      createEditor();
      }
      });

      function createEditor() {
      StackExchange.prepareEditor({
      heartbeatType: 'answer',
      autoActivateHeartbeat: false,
      convertImagesToLinks: true,
      noModals: true,
      showLowRepImageUploadWarning: true,
      reputationToPostImages: 10,
      bindNavPrevention: true,
      postfix: "",
      imageUploader: {
      brandingHtml: "Powered by u003ca class="icon-imgur-white" href="https://imgur.com/"u003eu003c/au003e",
      contentPolicyHtml: "User contributions licensed under u003ca href="https://creativecommons.org/licenses/by-sa/3.0/"u003ecc by-sa 3.0 with attribution requiredu003c/au003e u003ca href="https://stackoverflow.com/legal/content-policy"u003e(content policy)u003c/au003e",
      allowUrls: true
      },
      noCode: true, onDemand: true,
      discardSelector: ".discard-answer"
      ,immediatelyShowMarkdownHelp:true
      });


      }
      });














      draft saved

      draft discarded


















      StackExchange.ready(
      function () {
      StackExchange.openid.initPostLogin('.new-post-login', 'https%3a%2f%2fmath.stackexchange.com%2fquestions%2f525242%2fderive-error-term-by-using-taylor-series-expansions%23new-answer', 'question_page');
      }
      );

      Post as a guest















      Required, but never shown

























      2 Answers
      2






      active

      oldest

      votes








      2 Answers
      2






      active

      oldest

      votes









      active

      oldest

      votes






      active

      oldest

      votes









      1












      $begingroup$

      With the purpose of not leaving this question unanswered, this answer contains what OP already knows: stopping at $E_2$ is the cleanest way to go, since we already know how to express the error term of a Taylor series in convenient ways (Lagrange form, integral form). Some algebra quickly yields the error $h^{-2}(E_2(2h)-2E_2(h))$.






      share|cite|improve this answer









      $endgroup$


















        1












        $begingroup$

        With the purpose of not leaving this question unanswered, this answer contains what OP already knows: stopping at $E_2$ is the cleanest way to go, since we already know how to express the error term of a Taylor series in convenient ways (Lagrange form, integral form). Some algebra quickly yields the error $h^{-2}(E_2(2h)-2E_2(h))$.






        share|cite|improve this answer









        $endgroup$
















          1












          1








          1





          $begingroup$

          With the purpose of not leaving this question unanswered, this answer contains what OP already knows: stopping at $E_2$ is the cleanest way to go, since we already know how to express the error term of a Taylor series in convenient ways (Lagrange form, integral form). Some algebra quickly yields the error $h^{-2}(E_2(2h)-2E_2(h))$.






          share|cite|improve this answer









          $endgroup$



          With the purpose of not leaving this question unanswered, this answer contains what OP already knows: stopping at $E_2$ is the cleanest way to go, since we already know how to express the error term of a Taylor series in convenient ways (Lagrange form, integral form). Some algebra quickly yields the error $h^{-2}(E_2(2h)-2E_2(h))$.







          share|cite|improve this answer












          share|cite|improve this answer



          share|cite|improve this answer










          answered Oct 14 '13 at 17:26









          Ian MateusIan Mateus

          4,67032452




          4,67032452























              0












              $begingroup$

              As a central difference quotient for the position $x+h$, the Taylor expansion of the expression starts as
              $$
              frac1{h^{2}}[f(x)-2f(x+h)+f(x+2h)]=f''(x+h)+frac{h^2}{12}f^{(4)}(x+h)+O(h^4). tag{*}label{eq:1}
              $$



              By Taylor expansion of the second derivative $g(x)=f''(x)$ one gets that the deviation from $x$ to $x+h$ expands as
              begin{align}
              f''(x+h)=g(x+h)&=g(x)+g'(x)h+frac{h^2}2g''(x)+O(h^3)
              \
              &=f''(x)+hf'''(x)+frac{h^2}2f^{(4)}(x)+O(h^2). tag{**}label{eq:2}
              end{align}



              Inserting eqref{eq:2} into eqref{eq:1} gives in total
              $$
              frac1{h^{2}}[f(x)-2f(x+h)+f(x+2h)]=f''(x)+hf'''(x)+frac{7h^2}{12}f^{(4)}(x)+O(h^3),
              $$

              so that the error term from the question should be
              $$
              -hf'''(x)-frac{7h^2}{12}f^{(4)}(x)+O(h^3).
              $$

              Obviously the linear error term is the dominant one. So up to the missing $2$ in front of $E_3(h)=h^4R_3(h)$ as mentioned in the comments, the calculation is correct.






              share|cite|improve this answer











              $endgroup$


















                0












                $begingroup$

                As a central difference quotient for the position $x+h$, the Taylor expansion of the expression starts as
                $$
                frac1{h^{2}}[f(x)-2f(x+h)+f(x+2h)]=f''(x+h)+frac{h^2}{12}f^{(4)}(x+h)+O(h^4). tag{*}label{eq:1}
                $$



                By Taylor expansion of the second derivative $g(x)=f''(x)$ one gets that the deviation from $x$ to $x+h$ expands as
                begin{align}
                f''(x+h)=g(x+h)&=g(x)+g'(x)h+frac{h^2}2g''(x)+O(h^3)
                \
                &=f''(x)+hf'''(x)+frac{h^2}2f^{(4)}(x)+O(h^2). tag{**}label{eq:2}
                end{align}



                Inserting eqref{eq:2} into eqref{eq:1} gives in total
                $$
                frac1{h^{2}}[f(x)-2f(x+h)+f(x+2h)]=f''(x)+hf'''(x)+frac{7h^2}{12}f^{(4)}(x)+O(h^3),
                $$

                so that the error term from the question should be
                $$
                -hf'''(x)-frac{7h^2}{12}f^{(4)}(x)+O(h^3).
                $$

                Obviously the linear error term is the dominant one. So up to the missing $2$ in front of $E_3(h)=h^4R_3(h)$ as mentioned in the comments, the calculation is correct.






                share|cite|improve this answer











                $endgroup$
















                  0












                  0








                  0





                  $begingroup$

                  As a central difference quotient for the position $x+h$, the Taylor expansion of the expression starts as
                  $$
                  frac1{h^{2}}[f(x)-2f(x+h)+f(x+2h)]=f''(x+h)+frac{h^2}{12}f^{(4)}(x+h)+O(h^4). tag{*}label{eq:1}
                  $$



                  By Taylor expansion of the second derivative $g(x)=f''(x)$ one gets that the deviation from $x$ to $x+h$ expands as
                  begin{align}
                  f''(x+h)=g(x+h)&=g(x)+g'(x)h+frac{h^2}2g''(x)+O(h^3)
                  \
                  &=f''(x)+hf'''(x)+frac{h^2}2f^{(4)}(x)+O(h^2). tag{**}label{eq:2}
                  end{align}



                  Inserting eqref{eq:2} into eqref{eq:1} gives in total
                  $$
                  frac1{h^{2}}[f(x)-2f(x+h)+f(x+2h)]=f''(x)+hf'''(x)+frac{7h^2}{12}f^{(4)}(x)+O(h^3),
                  $$

                  so that the error term from the question should be
                  $$
                  -hf'''(x)-frac{7h^2}{12}f^{(4)}(x)+O(h^3).
                  $$

                  Obviously the linear error term is the dominant one. So up to the missing $2$ in front of $E_3(h)=h^4R_3(h)$ as mentioned in the comments, the calculation is correct.






                  share|cite|improve this answer











                  $endgroup$



                  As a central difference quotient for the position $x+h$, the Taylor expansion of the expression starts as
                  $$
                  frac1{h^{2}}[f(x)-2f(x+h)+f(x+2h)]=f''(x+h)+frac{h^2}{12}f^{(4)}(x+h)+O(h^4). tag{*}label{eq:1}
                  $$



                  By Taylor expansion of the second derivative $g(x)=f''(x)$ one gets that the deviation from $x$ to $x+h$ expands as
                  begin{align}
                  f''(x+h)=g(x+h)&=g(x)+g'(x)h+frac{h^2}2g''(x)+O(h^3)
                  \
                  &=f''(x)+hf'''(x)+frac{h^2}2f^{(4)}(x)+O(h^2). tag{**}label{eq:2}
                  end{align}



                  Inserting eqref{eq:2} into eqref{eq:1} gives in total
                  $$
                  frac1{h^{2}}[f(x)-2f(x+h)+f(x+2h)]=f''(x)+hf'''(x)+frac{7h^2}{12}f^{(4)}(x)+O(h^3),
                  $$

                  so that the error term from the question should be
                  $$
                  -hf'''(x)-frac{7h^2}{12}f^{(4)}(x)+O(h^3).
                  $$

                  Obviously the linear error term is the dominant one. So up to the missing $2$ in front of $E_3(h)=h^4R_3(h)$ as mentioned in the comments, the calculation is correct.







                  share|cite|improve this answer














                  share|cite|improve this answer



                  share|cite|improve this answer








                  edited Jan 8 at 18:57

























                  answered Aug 28 '17 at 12:58









                  LutzLLutzL

                  57.6k42054




                  57.6k42054






























                      draft saved

                      draft discarded




















































                      Thanks for contributing an answer to Mathematics Stack Exchange!


                      • Please be sure to answer the question. Provide details and share your research!

                      But avoid



                      • Asking for help, clarification, or responding to other answers.

                      • Making statements based on opinion; back them up with references or personal experience.


                      Use MathJax to format equations. MathJax reference.


                      To learn more, see our tips on writing great answers.




                      draft saved


                      draft discarded














                      StackExchange.ready(
                      function () {
                      StackExchange.openid.initPostLogin('.new-post-login', 'https%3a%2f%2fmath.stackexchange.com%2fquestions%2f525242%2fderive-error-term-by-using-taylor-series-expansions%23new-answer', 'question_page');
                      }
                      );

                      Post as a guest















                      Required, but never shown





















































                      Required, but never shown














                      Required, but never shown












                      Required, but never shown







                      Required, but never shown

































                      Required, but never shown














                      Required, but never shown












                      Required, but never shown







                      Required, but never shown







                      Popular posts from this blog

                      Can a sorcerer learn a 5th-level spell early by creating spell slots using the Font of Magic feature?

                      Does disintegrating a polymorphed enemy still kill it after the 2018 errata?

                      A Topological Invariant for $pi_3(U(n))$